It can be inferred from the passage that Garcia would most probably agree with which one of the following statements ...

on August 17, 2020

Why is A correct?

Can someone explain why A is the correct answer? (PT 74, S3, Q23)

Reply
Create a free account to read and take part in forum discussions.

Already have an account? log in

Shunhe on August 19, 2020

Hi @mb159701,

Thanks for the question! So we’re being asked here for something that Garcia would probably agree with concerning the Mexican American political activists of the 1930s and 1940s. Well, where does he talk about them? We’re told about this in the first paragraph, so that’s a good place to go looking for answers. And now take a look at lines 7-10, where we’re told that “Garcia does provide persuasive evidence that in the 1930s and 1940s thee activists anticipated many of the reforms proposed by the more militant Chicanos of the 1960s and 1970s.” So that suggests that he thought that their concerns were similar to Mexican American activists of the 1960s and 1970s, and that’s exactly what (A) tells us.

Hope this helps! Feel free to ask any other questions that you might have.